33bits
Pregunta logartimo neperiano. - Versión para impresión

+- 33bits (https://33bits.net/foro)
+-- Foro: Temas de debate (https://33bits.net/foro/Foro-Temas-de-debate)
+--- Foro: Off Topic (https://33bits.net/foro/Foro-Off-Topic)
+--- Tema: Pregunta logartimo neperiano. (/Tema-Pregunta-logartimo-neperiano)

Páginas: 1 2


Pregunta logartimo neperiano. - mike - 27-08-2014

Si tienes lo siguiente:

[tex](-1/3)*ln(-3-3y/x-3y^2/x^2)=ln(x) + c[/tex]

Como despejarías c eliminando los neperianos?

Jase debe saberlo seguro roto2cafe


Re:Pregunta logartimo neperiano. - snake_eater - 27-08-2014

Invoco a Jase tila


Re:Pregunta logartimo neperiano. - AcAnchoa - 27-08-2014

Lo único que puedes conseguir quitando neperianos ahí (para lo que hay recurrir al número e) es simplificar un poco la ecuación, pero al final tienes que volver a meter neperianos para despejar C.


Re:Pregunta logartimo neperiano. - mike - 27-08-2014

Es que me pone que la solución final sería:

[tex]c=y^3+x^2*y+x^3[/tex]

Y no se muy bien como lo ha hecho para llegar ahí.


Re:Pregunta logartimo neperiano. - Jase - 27-08-2014

Nop, yo de logaritmos no tengo ni idea XD
Sólo sé lo justo para resolver mi ejercicio de funciones sisi


Re:Pregunta logartimo neperiano. - wirmslayer - 27-08-2014

Son constantes distintas. Supongo que viene de una integración y la constante arbitaria c es eso, arbitaria. Despeja la ecuación y al final, a todas las constantes las agrupas en c y listo.

Si no lo ves, ahora edito con un desarrollo.

Edito:

[tex]-ln(-3-3y/x-3y^2/x^2) = 3*ln(x) + 3c[/tex]

[tex]-ln(-3-3y/x-3y^2/x^2) = ln(x^3)+ 3c[/tex]

[tex] -3c = ln(x^3) + ln(-3-3y/x-3y^2/x^2)[/tex]

[tex] -3c = ln(-3x^3 - 3yx^2 -3xy^2) [/tex]

[tex] e^(-3c) = -3*(x^3 + yx^2  + xy^2) [/tex]

[tex] x^3 + yx^2  + xy^2 = -e^(-3c)/3 = c' [/tex]

Revisa el paso intermedio, porque por la solucion que has puesto te has dejado una y en algun sitio.


Re:Pregunta logartimo neperiano. - mike - 27-08-2014

Exscto wirmslayer. Viene de una integracion.

Si me pones el desarrollo te lo agradeceria.


Re:Pregunta logartimo neperiano. - AcAnchoa - 27-08-2014

(27-08-2014 19:02)mike link escribió:Es que me pone que la solución final sería:

[tex]c=y^3+x^2*y+x^3[/tex]

Y no se muy bien como lo ha hecho para llegar ahí.
He llegado a:

[tex]C = y^{2}x + x^{2}y + x^{3}[/tex]


¿Seguro que es?

[tex]-\frac{1}{3} \ln{\left (-3 -3\frac{y}{x} -3\frac{y^2}{x^2} \right ) } = \ln{x} + C[/tex]


Si tuviese:

[tex]-\frac{1}{3} \ln{\left (-3 -3\frac{y}{x} -3\frac{y^3}{x^3} \right ) } = \ln{x} + C[/tex]


llegaría a tu solución. Me hago viejo, esto antes lo sacaba de corrido :'( .


Re:Pregunta logartimo neperiano. - mike - 27-08-2014

Es que sale eso al sustituir y=ux -> u=y/x

Entonces tienes -3-3u-3u^2.

Es una ec. diferencial homogenea.

Aunque es posible que el libro se haya equivocado, que puede ser.

Pd: gracias a ambos.


Re:Pregunta logartimo neperiano. - wirmslayer - 27-08-2014

¿Has sustituido tambien el diferencial dentro de la integral?

Suponiendo que la integral es en x

[tex]u=y/x[/tex]

Pero tambien

[tex] dx = d(y/u) = -y/u^2 * du[/tex]

No puedes hacer la integral en u con dx directamente, porque son variables distintas.


Re:Pregunta logartimo neperiano. - AcAnchoa - 27-08-2014

Si tuvieses

[tex]-\frac{1}{3} \ln{\left (-3 -3\frac{y}{x} -3\frac{y^3}{x^3} \right ) } = \ln{x} + c[/tex]


sería muy sencillito:




[tex]-\frac{1}{3} \ln{\left (-3 -3\frac{y}{x} -3\frac{y^3}{x^3} \right ) } = \ln{x} + c[/tex]


[tex]\ln{\left (-3 -3\frac{y}{x} -3\frac{y^3}{x^3} \right ) } = -3 \ln{x} -3c[/tex]


[tex]\ln{\left (-3 -3\frac{y}{x} -3\frac{y^3}{x^3} \right ) } + \ln{x^{3}} = -3c[/tex]


[tex]\ln{\left [ \left (-3 -3\frac{y}{x} -3\frac{y^3}{x^3} \right ) \cdot {x^{3}} \right ] = -3c}[/tex]


[tex]\ln{\left (-3 x^{3} -3x^{2}y -3y^{3} \right ) } = -3c[/tex]


[tex]e^{\ln{\left (-3 x^{3} -3x^{2}y -3y^{3} \right ) }} = e^{-3c}[/tex]


[tex]-3 x^{3} -3x^{2}y -3y^{3} = e^{-3c}[/tex]


[tex]-3(x^{3} + x^{2}y + y^{3}) = e^{-3c}[/tex]


[tex](x^{3} + x^{2}y + y^{3}) = -\frac{1}{3}e^{-3c}[/tex]


El término de la derecha es una constante, así que se reagrupa en C:

[tex]x^{3} + x^{2}y + y^{3} = C[/tex]



(27-08-2014 19:29)mike link escribió:Es que sale eso al sustituir y=ux -> u=y/x

Entonces tienes -3-3u-3u^2.

Es una ec. diferencial homogenea.

Aunque es posible que el libro se haya equivocado, que puede ser.

Pd: gracias a ambos.
Revisa la integral, que no puedes usar dx y es un fallo tonto pero que suele arruinarte varias horas.



Edito: Juirm, te odio, llevo un rato largo dándole al LaTeX y vas y te adelantas D:!


Re:Pregunta logartimo neperiano. - mike - 27-08-2014

Os dejo la integral.

[tex](3y^2+x^2)y'+2xy+3x^2=0[/tex]

Las he separado así:

[tex](3y^2+x^2)dy=(-2xy-3x^2)dx[/tex]

Hago la sustitución:

[tex]y=ux[/tex]
[tex]dy=udx+xdu[/tex]

Y llego hasta aquí:

[tex](3u+1)/(-3-3u-3^2)du=(1/x)dx[/tex]


No sé, lo volveré a revisar todo, a ver si se me ha escapado algo al sustituir y luego agrupar términos.